Solving a wave equation with seperation of variables.

Click For Summary
The discussion focuses on solving a wave equation using separation of variables, specifically the equation u_{tt} - 4u_{xx} = 4 with given boundary and initial conditions. The initial approach involved assuming a solution of the form u(x,t) = X(x)T(t), leading to a challenge in separating the variables due to the non-homogeneous term. A successful substitution, u(x,t) = v(x,t) - x^2/2, simplifies the equation to a homogeneous form, allowing for easier application of boundary conditions. The next step involves solving the homogeneous case for v(x,t) and then using the substitution to derive the solution for the original wave equation. The process emphasizes the importance of handling non-homogeneous terms effectively in partial differential equations.
Particle Head

Homework Statement


I am trying to solve the given wave equation using separation of variables,

u_{tt} - 4u_{xx} = 4 for 0 < x < 2 and t > 0
(BC) u(0,t) = 0 , u(2,t) = -2, for t>0
(IC) u(x,0)=x-x^2 , u_t(x,0)=0 for 0\leq x \leq2

Homework Equations


We are told we will need to use,

x = \frac{2L}{\pi} \sum_{n\geq1}^{} \frac{(-1)^{n+1}}{n} \sin{\frac{n\pi x}{L}}
x^2 = \frac{2L^2}{\pi} \sum_{n\geq1}^{} [\frac{(-1)^{n+1}}{n} + \frac{2}{n^3 \pi^2} ((-1)^n -1)] \sin{\frac{n\pi x}{L}}

The Attempt at a Solution


[/B]
I first assumed a solution of the form,

u(x,t) = X(x)T(t)

Plugging this back into the PDE this suggests that,

XT''-4X''T=4

With the homogeneous case we got a relation where in general \frac{T''}{c^2T} = \frac{X''}{X} = -\lambda and this is where I am unsure because I cannot seem to separate XT''-4X''T=4 in order to get a constant ratio between T and X.

I have a feeling I am supposed to solve the homogeneous case first however when progressing through that I ended up finding that \lambda = 0 satisfied my boundary conditions. This is because in the homogeneous case we want to solve X''+\lambda X = 0 and in the case where \lambda = 0 we have X = Ax+B and imposing the boundary conditions this seemed to imply X=-x

Just not sure how to go from here ?
 
Physics news on Phys.org
Can you find any other \lambda which satisfy the B.C.s?
 
Particle Head said:

Homework Statement


I am trying to solve the given wave equation using separation of variables,

u_{tt} - 4u_{xx} = 4 for 0 < x < 2 and t > 0
(BC) u(0,t) = 0 , u(2,t) = -2, for t>0
(IC) u(x,0)=x-x^2 , u_t(x,0)=0 for 0\leq x \leq2
Make the substitution ##u(x,t) = v(x,t) + \Psi(x)##. Put that into your equation and initial conditions and see if you can make a homogeneous problem in ##v(x,t)## by choosing ##\Psi(x)## in such a way to take care of the non-homogeneous terms in the DE and boundary conditions.
 
Thank you for your response,

I believe if I make the substitution u(x,t) = v(x,t) -\frac{x^2}{2} this enables me to take care of the non-homogeneous terms in the DE and BC. Substituting into the PDE I get v_{tt} - 4v_{xx} +4 = 4 \implies v_{tt} - 4v_{xx} =0 and the boundary conditions becomes u(0,t)=v(0,t)=0 and u(2,t) = v(2,t) -2 =- 2 \implies v(2,t) = 0

I presume from here we solve the homogeneous case in v(x,t) and then use our substitution to get the solution for the original PDE.
 
Particle Head said:
Thank you for your response,

I believe if I make the substitution u(x,t) = v(x,t) -\frac{x^2}{2} this enables me to take care of the non-homogeneous terms in the DE and BC. Substituting into the PDE I get v_{tt} - 4v_{xx} +4 = 4 \implies v_{tt} - 4v_{xx} =0 and the boundary conditions becomes u(0,t)=v(0,t)=0 and u(2,t) = v(2,t) -2 =- 2 \implies v(2,t) = 0

I presume from here we solve the homogeneous case in v(x,t) and then use our substitution to get the solution for the original PDE.

Yes, that looks good. Don't overlook that fact that the initial condition now becomes ##v(x,0) = u(x,0)-\Psi(x)##.
 
Question: A clock's minute hand has length 4 and its hour hand has length 3. What is the distance between the tips at the moment when it is increasing most rapidly?(Putnam Exam Question) Answer: Making assumption that both the hands moves at constant angular velocities, the answer is ## \sqrt{7} .## But don't you think this assumption is somewhat doubtful and wrong?

Similar threads

  • · Replies 11 ·
Replies
11
Views
3K
Replies
6
Views
2K
Replies
1
Views
2K
  • · Replies 11 ·
Replies
11
Views
2K
  • · Replies 5 ·
Replies
5
Views
2K
  • · Replies 7 ·
Replies
7
Views
2K
Replies
1
Views
2K
  • · Replies 1 ·
Replies
1
Views
2K
  • · Replies 6 ·
Replies
6
Views
1K
Replies
9
Views
2K